8

Let $(\Omega, \mathscr F, \mathbb P)$ be a probability space. Conjecture:

Suppose we have events $A_1, A_2, ...$ s.t. $\forall \ A \in \bigcap_n \sigma(A_n, A_{n+1}, ...)$, $P(A) = 0$ or $1$. There exists an independent sequence of events $B_1, B_2, ...$ s.t.

$$\tau_{A_n} := \bigcap_n \sigma(A_n, A_{n+1}, ...) = \bigcap_n \sigma(B_n, B_{n+1}, ...) := \tau_{B_n}$$

Is this true?


I think there exists a function $f: \mathbb N \to \mathbb N$ s.t. $A_{f(n)}$'s are independent so we can choose $B_n = A_{f(n)}$. Is that true? Why/Why not? If not, how else can I prove or disprove the conjecture above? If it is true, I think it can be proven by modifying the proof of the Kolmogorov 0-1 Law (for events).


Perhaps one of these subsequences of sets is independent:

$$A_n$$

$$A_{2n}, A_{2n+1}$$

$$A_{3n}, A_{3n+1}, A_{3n+2}$$

$$\vdots$$

$$A_{mn}, A_{mn+1}, A_{mn+2}, ..., A_{mn+(m-1)}$$

$$\vdots$$

I think we have that

$$\tau_{A_n} = \tau_{A_{mn+i}} := \bigcap_n \sigma(A_{mn+i}, A_{m(n+1)+i}, ...)$$

where $m \in \mathbb N$ and $i \in \{0, 1, 2, ..., m-1\}$.


It seems like we need any such $f(n)$, if it exists, to satisfy the following condition:

$$\sigma(A_{f(n)}, A_{f(n+1)}...) \subseteq \sigma(A_n, A_{n+1}, ...) \tag{**}$$

which I guess is true if (and only if?) $f(n) \ge n$.


Other possible candidates for $f(n)$: (assume the variables are s.t. $f: \mathbb N \to \mathbb N$ is satisfied. If need be, $(**)$ or $f(n) \ge n$ too.)

  1. $\sum_{i=0}^{m} a_i n^i$

  2. $2^n, 3^n, ...$

  3. $\sum_{i=1}^{m} b_i c_i^n$

  4. $\lfloor{t^n}\rfloor, \lceil{t^n}\rceil$ (I guess $t > e^{1/e}$)

  5. $\lfloor{\sum_{i=1}^{m} b_i c_i^n}\rfloor, \lceil{\sum_{i=1}^{m} b_i c_i^n}\rceil$

  6. $\lfloor{\text{linear combination of trigonometric functions}}\rfloor, \lceil{\text{linear combination of trigonometric functions}}\rceil$

  7. $\lfloor{\text{Some linear combination of the above}}\rfloor, \lceil{\text{Some linear combination of the above}}\rceil$


Assuming the conjecture is true, I guess it's not necessary to find $f(n)$ that works for all possible sequences of events $A_1, A_2, ...$ because such $f(n)$ may not even exist.


To disprove the conjecture: I guess we must show that such a sequence $B_n$ being independent implies $B_n$ tail will never equal $A_n$ tail since $B_n$ tail will be $\mathbb P-$trivial by Kolmogorov 0-1 Law (for events).

Something that might help: we could show that $\forall \ A \in \bigcap_n \sigma(A_{f(n)}, A_{f(n+1)}, ...), P(A) = 0$ or $1$ and $\forall n \in \mathbb N, A_{f(n)}, A_{f(n+1)}, ...$ is not independent, but I'm not quite sure that the conjecture is disproved because we could construct some $B_n$'s that look like:

  1. $$B_n = A_{n+1} \setminus A_n$$

  2. $$B_n = A_{n} \setminus A_{n-1}, A_0 = \emptyset$$

  3. $$B_n = \bigcap_m A_{mn}$$

  4. $$B_n = \bigcup_m A_{mn}$$

  5. $$B_{2n} = \bigcap_m A_{mn}, B_{2n+1} = \bigcup_m A_{mn}$$

  6. $$B_n = \limsup_m A_{mn}$$

  7. $$B_n = \liminf_m A_{mn}$$

  8. $$B_{2n} = \limsup_m A_{mn}, B_{2n+1} = \liminf_m A_{mn}$$

Not to say of course that any of those $B_n$'s satisfy $\tau_{A_n} = \tau_{B_n}$ but that $B_n$ need not be in the form $A_{f(n)}$.


Borel-Cantelli:

  1. If $\sum_n P(A_n) < \infty \to 0 = P(\limsup A_n) = P(\limsup A_{mn}) \ \forall m \in \mathbb N$. Hence $B_m = \limsup A_{mn}$ is independent.

  2. If $\sum_n P(A_n) = \infty$, then maybe this extension of Borel-Cantelli? Not quite sure I understand it or how it would be helpful. I don't think we can conclude anything if we have $P(\limsup A_n)$.

  3. Then there's the case of $\sum_n P(A_n) = \infty$ but the conditions earlier aren't satisfied.

BCLC
  • 2,166
  • 2
  • 22
  • 47
  • 1
    Perhaps a proof by construction, where $B_1 = A_1, B_2 = A_2 - A_1, \dots$? – jbowman Dec 24 '15 at 01:55
  • @jbowman Well those are pairwise disjoint, I think. Are they independent? I was thinking about shifting the indices instead of constructing a pairwise disjoint collection – BCLC Dec 24 '15 at 02:15
  • @jbowman Thanks for the suggestion, but I don't think those events are even pairwise indp. Intersection of any two is empty set. I don't think zero is the product of the probabilities of the two. Wait do you mean to use that disprove the conjecture? – BCLC Dec 24 '15 at 05:56
  • 1
    To me this conjecture seems unlikely to be true unless you add extra conditions, or you mean that the two $\sigma$-algebra's completions agree (which holds almost trivially). However I can't see a counter example. – P.Windridge Dec 25 '15 at 14:20
  • @P.Windridge What do you mean completions agree? Must we really find a counter example to disprove this conjecture? It seems to me that we have to assume there exists such a sequence and then show that there is a contradiction. To prove the conjecture, we must show only a single sequence of events that satisfies the conditions – BCLC Dec 25 '15 at 17:33
  • Completion of a $\sigma$-algebra (relative to a probability measure) = add all subsets null sets. – P.Windridge Dec 27 '15 at 10:17
  • "It seems to me that we have to assume there exists such a sequence and then show that there is a contradiction": well, your assertion (that a given $\mathbb{P}$-trivial tail $\sigma$-algebra is equal to the tail $\sigma$-algebra of some independent events) is true in many cases, so there wouldn't be a contradiction. So a special example is needed. But perhaps I misunderstood what you are asking. – P.Windridge Dec 27 '15 at 10:21
  • "To prove the conjecture, we must show only a single sequence of events": you mean give a single construction of the events $B_n$? – P.Windridge Dec 27 '15 at 10:24
  • In your question, you have "I think there exists a function $f: \mathbb N \to \mathbb N$ s.t. $A_{f(n)}$'s are independent so we can choose $B_n = A_{f(n)}$". If $f$ is deterministic (as you try to construct in the question), then this is just changing the labels. But if you have a bunch of **dependent** events, then just changing their labels cannot make them not dependent :) – P.Windridge Dec 27 '15 at 10:28
  • I should explain my skepticism about the conjecture: if a $\sigma$-algebra is countably generated (as the tail $\sigma$-algebra of $A_1,A_2,\ldots$ can be if you allow the $A_i$ to be dependent) then it is atomic. [Citation needed]. If there is $0-1$ valued measured on such a countably generated $\sigma$-algebra then there is necessarily an atom of mass $1$ [Citation needed]. On the other hand I'm pretty sure you can cook up probability spaces such that the tail $\sigma$-algebra for any sequence of *independent* events has no atom of mass $1$. – P.Windridge Dec 27 '15 at 10:42
  • 1
    In any case I think you can start with the (simpler) question: "Let $(\Omega, \mathcal{F}, \mathbb{P})$ be a probability space. Suppose $\mathcal{G}\subset\mathcal{F}$ is a countably generated $\sigma$-algebra and that $\mathbb{P}(A)= 0$ or $1$ for any event $A \in \mathcal{G}$. Is there a sequence of independent events $B_1,B_2,\ldots$ in $\mathcal{F}$ with tail $\sigma$-algebra $\mathcal{G}$? – P.Windridge Dec 27 '15 at 10:51
  • @P.Windridge Thanks for the comment. I'm in the process of digesting them. Re [this comment](http://stats.stackexchange.com/questions/187989/conjecture-related-to-kolmogorov-0-1-law-for-events?noredirect=1#comment357500_187989), what exactly do you mean? In the link I gave, the $A_n$'s are not independent but $A_{mn}$'s are independent. – BCLC Dec 27 '15 at 10:53
  • @P.Windridge Interesting revision. I'll add that. Thanks ^-^ – BCLC Dec 27 '15 at 10:54
  • @P.Windridge On second thought, what's the difference? I think $\mathcal G$ is countably generated iff there exists some events $A_1, A_2, ...$ s.t. $\bigcap_n \sigma (A_n, A_{n+1}, ...) = \mathcal G$. – BCLC Dec 27 '15 at 10:58
  • For a general sequence of dependent events $A_1,\ldots$, there is a no rearrangement of the labels that will make them independent! – P.Windridge Dec 27 '15 at 11:02
  • 2
    A $\sigma$-algebra $\mathcal{G}$ is countably generated if there exists $F_1,F_2,\ldots$ s.t. $\mathcal{G} = \sigma(F_1,F_2,\ldots)$. It is straightforwards to find examples where the tail $\sigma$-algebra is *not* countably generated. – P.Windridge Dec 27 '15 at 11:03
  • 2
    More generally, a sub-$\sigma$-algebra of a countably generated $\sigma$-algebra may not itself be countably generated! Actually look at Exercise 1.1.18 in http://math.mit.edu/~dws/175/prob01.pdf – P.Windridge Dec 27 '15 at 11:06
  • 1
    That exercise makes a good point that I overlooked- you might want to first require that $\mathbb{P}(B_i)$ is bounded away from $0$ or $1$ (to rule out a boring sequence of events which are independent only by virtue of all being null etc). – P.Windridge Dec 27 '15 at 11:14
  • @P.Windridge Did you perhaps mean to say $\mathcal G \subseteq \mathcal F$ where $\mathcal G$ may or may not be countably generated? – BCLC Dec 27 '15 at 11:28
  • @P.Windridge I was thinking to require that $0 < P(B_i) < 1$, but if not, would we really have $(*)$? – BCLC Dec 27 '15 at 11:29
  • 1
    You could indeed consider $\mathcal{G}$ not countably generated but I was suggesting you initially restrict attention to the simpler case that $\mathcal{G}$ is countably generated. – P.Windridge Dec 27 '15 at 11:32
  • @P.Windridge How am I not restricting myself to such $\mathcal G$? – BCLC Dec 27 '15 at 11:34
  • 1
    A common strategy in maths is to add simplifying assumptions to give a question you can answer. Then you can try to relax the assumptions. – P.Windridge Dec 27 '15 at 11:34
  • 1
    because you said "[is] **or may not be** countably generated" – P.Windridge Dec 27 '15 at 11:36
  • @P.Windridge I don't quite understand. You suggested to revise my conjecture to what you said. How is my conjecture different from what you said? It seems to me that they are equivalent. – BCLC Dec 27 '15 at 11:38
  • 1
    I made the additional assumption that $\mathcal{G} = \bigcap_n \sigma(A_n, A_{n+1}, ...)$ is countably generated, that's all. – P.Windridge Dec 27 '15 at 11:45
  • @P.Windridge Ah, my mistake. I thought $\mathcal G$ was countably generated by definition. Thanks ^-^ – BCLC Dec 27 '15 at 11:46
  • @P.Windridge [Rearrangement](http://stats.stackexchange.com/questions/187989/conjecture-related-to-kolmogorov-0-1-law-for-events?noredirect=1#comment357513_187989)? Just because $A_1, A_2, ...$ is not an independent sequence doesn't mean $A_2, A_4, ...$ can't be an independent sequence – BCLC Dec 27 '15 at 11:47
  • Indeed. But your conjecture is stated for arbitrary sequences of events. In particular, I can take $A_i = A$ for some event with $\mathbb{P}(A) \in (0,1)$ and then **no** rearrangement or sub-sequence of the $A_i$ is independent! – P.Windridge Dec 27 '15 at 11:52
  • @P.Windridge 1 If you mean $P(A) \in (0,1)$, then how does that satisfy the assumption that $P(B) = 0$ or $1$ for $B \in \bigcap_n \sigma(A_n, A_{n+1}, ...) = \bigcap_n \sigma(A) = \sigma(A) = \{\emptyset, \Omega, A, A^C\}$? 2 If you mean $P(A) \in \{0,1\}$, then [$A$ is independent of all events, including itself](http://stats.stackexchange.com/questions/186619/does-an-unconditional-probability-of-1-or-0-imply-a-conditional-probability-of-1)...I think? – BCLC Dec 27 '15 at 12:02
  • 1
    The $\mathbb{P}(A) \in (0,1)$ was intentional. Indeed, $A_i = A$ then does not have $\mathbb{P}$-trivial tail $\sigma$-algebra. However, the point was to illustrate that taking subsequences or relabelling dependent events cannot in general give you independence. (Perhaps it would be a useful exercise to construct a sequence $A_i$ of dependent events that *does* have a $\mathbb{P}$-trivial tail $\sigma$-algebra). – P.Windridge Dec 27 '15 at 13:17
  • @P.Windridge Well yeah. In what I linked above, we are given that the events are independent for a certain distance between indices. Here we aren't. However, we are given the assumption of a trivial tail. That might mean something. If polynomial functions don't work, perhaps some other kind of $f(n)$ (maybe involving floor/ceiling functions). I tried looking up that and found nothing. Hints please? ^-^ – BCLC Dec 28 '15 at 04:15
  • 1
    Cross-posted at https://math.stackexchange.com/q/1615954/59351. – Scortchi - Reinstate Monica Apr 24 '18 at 09:20

1 Answers1

7

If you want events $B_n$ that are independent in an interesting manner (not simply because $\mathbb{P}(B_n) = 0$ or $\mathbb{P}(B_n) = 1$) then the conjecture is false.

Here is a pedantic example. Suppose $(\Omega, \mathcal{F},\mathbb{P})$ is a suitably rich probability space.

Let $A \in \mathcal{F}$ be $\mathbb{P}$-null, i.e. $\mathbb{P}(A) =0$. Take $A_i = A$, so that the tail $\sigma$-algebra is $\mathcal{G} = \{\emptyset, A, A^c, \Omega\}$.

Note that in particular $\mathcal{G}$ is finite.

Now, suppose that $B_1, B_2, \ldots$ is an independent sequence of events with $\mathbb{P}(B_n)$ bounded away from $0$ and $1$. Then the tail $\sigma$-algebra $\mathcal{H}$ is not countably generated. (See e.g. Exercise 1.1.18 http://math.mit.edu/~dws/175/prob01.pdf, which uses an argument like I outlined above- any countably generated $\mathbb{P}$-trivial $\sigma$-algebra has an atom of mass $1$, but $\mathcal{H}$ has no such atom).

So, $\mathcal{G}$ is finite but $\mathcal{H}$ is not even countably generated.


Edit 2: if you accept $\mathbb{P}(B_n) = 0$ then you can replicate any countably generated $\mathbb{P}$-trivial $\sigma$-algebra. In more detail, suppose that $\mathcal{G}$ is generated by events $E_1, E_2, \ldots \in \mathcal{G}\subset\mathcal{F}$. If $\mathcal{G}$ is $\mathbb{P}$-trivial then the $E_n$ are all independent, by virtue of being null (or $E_n^c$ being null). Now make a triangular construction for the $B$ events: $B_{1,1} = E_1$, $B_{2,1} = E_1, B_{2,2} = E_2,\ldots,B_{k,j} = E_j$, $1\le j \le k$.

Then $(B_{k,j})$ is a countable sequence (with natural ordering for the indices) of independent events whose tail $\sigma$-algebra is $\mathcal{G}$.

So, here I think is the key question: suppose that $\mathcal{G}$ is a non-countably-generated $\mathbb{P}$-trivial tail $\sigma$-algebra (coming from non-null events which might be dependent). Can $\mathcal{G}$ be realised as the tail $\sigma$-algebra for some null events?

Edit 1: A gray area is what happens if you accept $\mathbb{P}(B_n)\to 0$, although that doesn't seem to be the thrust of the original question.

P.Windridge
  • 2,075
  • 12
  • 13
  • Thanks P.Windridge, but I'm not quite sure I understand. 1 If we include $P(B_n)=0$ or $1$, the conjecture is (trivially?) true? 2 Is what you're trying to prove in Edit 2? If so, is your $\mathcal G$ equal to my $\tau_{A_n}$? I edited OP for shorthand – BCLC Dec 30 '15 at 22:30
  • I read the exercise. $\mathcal H = \tau_{B_n}$ ? – BCLC Dec 30 '15 at 22:49
  • Hi BCLC, (1) I'm saying that if we include $P(B_n)=0$ then the conjecture is true for all choices of the events $A_1,A_2,\ldots$ that have a "nice" tail $\sigma$-algebra (where "nice" here means countably generated). (2) Yes $\mathcal{G}=\tau_{(A_n)}$ and $\mathcal{H}$ is your $\tau_{(B_n)}$. *N.B.* the linked exercise uses "$A_n$" for what-should-be your $B_n$, and "$B_n$" denotes a candidate sequence of generating events (used to get a contradiction. – P.Windridge Dec 31 '15 at 20:05
  • I'm not sure I follow. Just from the assumptions, the $A_i$'s ARE independent? – BCLC Jan 01 '16 at 02:36
  • In Exercise 1.1.18 of http://math.mit.edu/~dws/175/prob01.pdf, the $A_i$'s are independent events, which you should think of as the $B_i$'s in your conjecture. Is that what you were asking? – P.Windridge Jan 01 '16 at 10:31
  • I mean the A_i's in my question. Your Edit 2 seems to suggest that we can conclude that they are independent – BCLC Jan 01 '16 at 21:42
  • How does the edit suggest that the $A_i$ are independent? Note that the $E_i$ events aren't the $A_i$, and dependent events $A_i$ can still generate a $\mathbb{P}$-trivial $\sigma$-algebra. – P.Windridge Jan 02 '16 at 08:49
  • Wait, I think I understand now: 1 If we allow $P(B_n)=0$ or $1$, the conjecture is true? 2 Otherwise, the conjecture is false (To prove such we assume such an independent sequence exists and then show it is not countably generated) ? – BCLC Jan 02 '16 at 16:07
  • Reread exercise. $\mathcal T = \bigcap_{n} \mathcal F_n$ where $\mathcal F_n = \sigma(A_n, A_{n+1}, ...)$? I'm not quite sure how $P(C) = 0$. I'll reread again later. So yeah, if such is really the case, that disproves the conjecture for $0 < P(B_n) < 1$? Wait so how does allowing $P(B_n) = 0$ prove the conjecture? I understand why the argument in the 1.1.18 doesn't disprove the conjecture, but I don't quite see why the conjecture is true – BCLC Jan 02 '16 at 16:14
  • Allowing $P(B_n) = 0$ doesn't make the conjecture true for all choices of the original events $(A_n)$, but it at least means the conjecture is true for "nice" choices. – P.Windridge Jan 02 '16 at 19:44
  • OK, about $P(C) =0$: can you see why $\mathbb{P}(\cap_{n\ge m} C_n) = 0$ (where $C_n$ are independent with probability bounded below $1$)? What do you know about countable unions of null events? – P.Windridge Jan 02 '16 at 19:47
  • Strictly speaking, the exercise concerns $\alpha \le \mathbb{P}(B_n) \le 1-\alpha$, for some $\alpha > 0$ (however small). (Can you see what goes wrong in the exercise when $\mathbb{P}(B_n) \to 0$?). – P.Windridge Jan 02 '16 at 19:50
  • So what's a counterexample of a choice of An s.t. there do not exist such Bn's? – BCLC Jan 04 '16 at 05:53
  • My answer gives an example for $\mathbb{P}(B_n)$ bounded away from 0 and 1. I still haven't thought of an example where $\tau(A_n)$ is not countably generated and $\mathbb{P}(B_n)$ is 0 or 1. – P.Windridge Jan 09 '16 at 14:45
  • Reread. Let's see if I understand. Case 1: If $\forall n, \ P(B_n) \notin \{0,1\}$ then, the conjecture is false. So for any hope for the conjecture to be true, we must have Case 2: there exists a $P(B_n) \in \{0,1\}$ (*). Now, we have two subcases. Case 2 Subcase 1: $\tau_{\{A_n\}}$ is countably generated. The conjecture is true for Case 2 Subcase 1. Case 2 Subcase 2: Not countably generated --> Still unknown. Is that right? – BCLC May 03 '18 at 07:09
  • (*) Please clarify. Do you indeed mean to say $\exists$ rather than $\forall n, P(B_n) \in \{0,1\}$ ? – BCLC May 03 '18 at 07:10
  • What exactly do you mean by triangular construction? I mean, what are the B_n's exactly? I guess it's something like $$\bigcap_{n=1}^{\infty} \sigma(\bigcup_{k=1}^{\infty} \sigma(B_{n,k})) = \bigcap_{n=1}^{\infty} \sigma(\bigcup_{k=1}^{\infty} \sigma(E_k))$$ where $B_{n,k} = E_k$ for $k \le n$ and $B_{n,k} = \Omega$ for $k > n$ – BCLC May 03 '18 at 10:16
  • P.Windridge, do you agree to [reject the edit](https://stats.stackexchange.com/review/suggested-edits/186316)? – BCLC May 03 '18 at 14:48
  • Hi @BCLC I'm afraid so and frankly I'm not sure why Xi'an initially accepted it, since it appears irrelevant. – P.Windridge May 03 '18 at 15:52
  • In the part you starred all events have probability $0$ or $1$, so no I don't mean "$\exists$", (More generally I can only see one minor typo in the answer that I don't intend to edit...) – P.Windridge May 03 '18 at 15:54
  • huh? So there are actually more than just those 2 cases then? Hmmm... – BCLC May 03 '18 at 16:01
  • 'Triangular' here means $B_{k,j}$ is not defined for $j > k$, so no $B_{n,k}$ is not $\Omega$. – P.Windridge May 03 '18 at 16:02
  • so what exactly are the Bn's please? I don't see what they are given the double indexed B's... – BCLC May 03 '18 at 16:41
  • Imagine that the $B_{k,j}$ are arranged like a triangle, with $B_{1,1}$ at the top in the first row, then $B_{2,1},B_{2,2}$ on the second row, etc. Now read them off top to bottom, left to right, so the sequence is $(B_{1,1}, B_{2,1},B_{2,2}, B_{3,1}, B_{3,2},B_{3,3}, \ldots)$ – P.Windridge May 03 '18 at 19:32
  • so the sequence is E1 E1 E2 E1 E2 E3 E1...? What's the explicit formula for the single index B's in terms of the double index B's? Is this perhaps like with proving the positive rationals are countable? – BCLC May 04 '18 at 05:25